Wenn ich nicht zocke dann...


Lockenlord said:
Wo das Problem herkommt ist ja egal. In meinem isolierten Ausschnitt der Schaltung ergibt sich ein Widerstandswert zwischen zwei nebeneinander liegenden Knoten aus R=3/(3+n) Ohm. Fün n gegen unendlich geht also der Widerstand gegen null. Der Widerstand im gesamten Netzwerk ist durch die zusätzlichen Parallelschaltungen mit weiteren Widerständen für alle n kleiner als in meinem Ausschnitt, also geht der Widerstand zwischen zwei benachbarten Knoten gegen null, womit auch der Widerstand zwischen zwei beliebigen Knoten null ist.

Prove me wrong. ;)

*FOOOOOOOOOOOOOOOOOMMMMMMMMMMMMMMMMM* :lol:

btt:

...das ist die meiste Zeit, warte schon sehnsüchtig auf meine Pandora :)...
...dann unternehm ich was mit meiner Freundin/Freunden...
...bin bei den Pfadfindern...
...gammel mal wieder am PC rum...
...brüte komische Ideen aus...
...und was man halt sonst noch so macht ;)
 
...bin ich auch bei den Pfadfindern (habe ich aber schon geschrieben)
... übe ich E-Gitarre (bin aber noch nicht viel weitergekommen, ich sollte mal nach System üben anstatt nur zu klimpern und mit den Effekten rumzuspielen, oder Lieder aus meinen 3Dicken Liederbüchern "Das Ding (1 2 3)" zu schrammeln..
.... bin ich noch bis Ende November als Aushilfe im Archiv der Klinik in Saarbrücken
... kaufe ich was zu zocken, heute: God of War Ghost of Sparta für die PSP und James Bond 007 Goldeneye für den Nintendo DS
.... Lese ich Zeitungen, N-Zone, Welt der Wunder, PM, Bücher (momentan Terry Pratschett: "Klonk"
... esse ich viel zu viel Schockolade

.... Außerdem habe ich seit langem nichts mehr gezockt, ich sollte weitermachen..

.... Warte ich auf meine Pandora (3265) und mittlerweile auf den Realease des Nintendo 3DS (in Blau)
..... Warte ich auf die Geburt meiner Nichte im Dezember

.... Träume ich schon das ich meine Pandora schon hätte (das ist echt seltsam)
 
OK, was ich heute Vormittag geschrieben habe war Blödsinn. Ich habe es noch nicht in eine Formel gepackt, aber der Ersatzwiderstand zwischen zwei direkt benachbarten Knoten geht in eine Richtung gegen 2/e Ohm. Das kann man aber nur näherungsweise für Widerstände annehmen, die nicht auf dem kürzesten Weg zwischen den beiden Knoten liegen. Der näherungsweise Widerstandswert zwischen den beiden gegebenen Knoten ist damit [4/(e+2)]||(e+4)/(e+2)=[(e+2)/4+(e+2)/(e+4)]^-1 Ohm, also etwa 531,385 mOhm. Wieviele Laster waren das jetzt? :-D

OK, zum Thema: Wenn ich nicht zocke, dann
... denke ich mir frivole Nicknames aus und schreibe Trollkommentare auf PandoraToday.
... koche ich was leckeres.
... trinke ich guten Espresso.
... gehe ich studieren (und rechne dabei auch wieder irgendwelche Widerstandsnetzwerke durch).
... warte ich auf meine Pandora, oder nee, halt, das mache ich auch während ich zocke.
... spiele ich Gitarre und singe dazu.
 
...gehe ich arbeiten
...fahre ich durch die Weltgeschichte...um zu arbeiten
...unternehme ich was mit meiner Freundin
...kaufe ich billig Waren und verkaufe diese wieder teuer über das Internet...oft mit meiner Freundin zusammen
...lese ich Bücher oder höre Audioversionen von ihnen, wenn ich mal meine Augen zu machen möchte dabei
...kümmer ich mich um 5 Hamster die in großen Aquarien leben
...lese ich Nachrichten über das Internet
...schaue ich Serien, Dokus, Filme am Notebook/Pandora oder am TV (aber kein normales Fernsehen, ist mir zu 90% zu dämlich geworden)
...bin ich einfach faul und schlafe...
...erledige ich Hausarbeit
...reinige den Kaminofen
...hacke ich Holz
...kümmere ich mich um unsere 10 Hühner hinterm Haus
...gehe ich mit meiner Freundin shoppen
...fülle ich die Druckerpatronen auf
...räume ich den Keller auf
...mecker ich meine Freundin voll weilse wieder alles unordentlich gemacht hat
...höre ich in ruhe Musik
...trinke ich Cuba Libre
...gehe ich mit zu Geschäftsessen und Seminaren, wo es auch wieder Essen gibt ^^
...mähe ich den Rasen

hm mir würde nochmehr einfallen nur ich lass es lieber^^
 
Christoph.K said:
Lies dir den Tooltip durch, den der Comic auf der xkcd-Seite hat. Dann wirst du feststellen, dass die Frage dem "Google Labs Aptitude Test" entstammt. So, und mit diesen Worten wünsche ich dir jetzt viel Spaß! ;)












..."There is nothing that could possibly improve upon emptiness, since it enables one to create something better..." oder Zirkulärreferenz... "1" wäre technisch gesehen eigentlich korrekt, zum Teil Verwaltungsoverhea... äh, aber sofern jemand auf Ideen kommt, die der andere allein nicht gehabt hätte, dann... Gelb, Lila... hm, erst mal recherchieren, was es da für Studien dazu... äh, hey! Was machst du da?!? Ich hab hier gerade zu tun, also lies gefälligst woanders weiter, ja? Pff, unhöflich, diese LeuWROOOOOOOM
Lockenlord said:
Wo das Problem herkommt ist ja egal.
Ich wollte damit sagen, dass in dem Test natürlich noch mehr mindestens interessante Aufgaben stecken. So z.B. "This space is intentionally blank. Please fill it with something that improves upon emptiness.". Aber auch greifbarere, wie z.B. "
Given a triangle ABC, how would you use only a compass and straight edge to find a point P such that triangles ABP, ACP, and BCP have equal perimeters? (Assume that ABC is constructed so that a solution does exist.)"

Ich finde solche Tests immer unterhaltsam und entspannend, daher wollte ich dieses Detail nicht unerwähnt lassen.


------------------------------------------------------------------------
Lockenlord said:
OK, was ich heute Vormittag geschrieben habe war Blödsinn. Ich habe es noch nicht in eine Formel gepackt, aber der Ersatzwiderstand zwischen zwei direkt benachbarten Knoten geht in eine Richtung gegen 2/e Ohm. Das kann man aber nur näherungsweise für Widerstände annehmen, die nicht auf dem kürzesten Weg zwischen den beiden Knoten liegen. Der näherungsweise Widerstandswert zwischen den beiden gegebenen Knoten ist damit [4/(e+2)]||(e+4)/(e+2)=[(e+2)/4+(e+2)/(e+4)]^-1 Ohm, also etwa 531,385 mOhm. Wieviele Laster waren das jetzt? :-D
Wartet (ohne die Lösung zu kennen) schon jetzt gespannt auf die Fortsetzung, da eine Antwort welche das Wort "näherungsweise" enthält vermutlich nicht die gesuchte ist,

Christoph :)
 
Christoph.K said:
Wartet (ohne die Lösung zu kennen) schon jetzt gespannt auf die Fortsetzung, da eine Antwort welche das Wort "näherungsweise" enthält vermutlich nicht die gesuchte ist,

Christoph :)
Der Widerstandswert in einem endlichen Ausschnitt nähert sich ziemlich schnell dem theoretischen Wert eines unendlichen Gitters. Das Problem ist, dass ich das ganze noch nicht in eine Formel getan habe und man von außen nach innen rechnen muss. Ich muss also jedes mal wieder von vorne beginnen, wenn ich eine Lage Widerstände hinzufüge und mit jedem Durchgang steigt der Rechenaufwand exponentiell. 2/e stimmt auch schon nach der dritten Nachkommastelle nicht mehr, sehe ich gerade. :dash:
Christoph.K said:
Ich wollte damit sagen, dass in dem Test natürlich noch mehr mindestens interessante Aufgaben stecken.
Argh, nein, nicht noch mehr. Die Probleme sind zwar durchaus interessant und wahrscheinlich auch echte Herausforderungen, aber wenn ich das Widerstandsnetzwerk geknackt habe habe ich noch ein Kilo E-Technik-Übungsaufgaben, das ich bis zu den Prüfungen durchrechnen will, auch wenn es wahrscheinlich nicht halbsoviel Zeit verschlingen wird.
 
Natürlich vom zocken und tippen, was dachtet ihr denn? :O ;)
 
Back
Top